If f(x) and g(x) are both continue fungtions such that f(g(x))=x and g(f(x) )=x, f(0)=0,f(3)=29. find ∫_0^3▒f(x)dx + ∫_0^29▒g(x)dx.
Find the area of a region bounded by the curve y=1/(1+x^2 ) , x positive axis and y positive axis.
Easy Math Editor
This discussion board is a place to discuss our Daily Challenges and the math and science related to those challenges. Explanations are more than just a solution — they should explain the steps and thinking strategies that you used to obtain the solution. Comments should further the discussion of math and science.
When posting on Brilliant:
*italics*
or_italics_
**bold**
or__bold__
paragraph 1
paragraph 2
[example link](https://brilliant.org)
> This is a quote
\(
...\)
or\[
...\]
to ensure proper formatting.2 \times 3
2^{34}
a_{i-1}
\frac{2}{3}
\sqrt{2}
\sum_{i=1}^3
\sin \theta
\boxed{123}
Comments
integral 0 to 29 g(x)dx= integral 0 to 3 g(f(t))f'(t)dt,
so total integral becomes integral 0 to 3 ( f(t)dt + g(f(t)f'(t)dt) but g(f(t))= t,
so integral = integral 0 to 3 (f(t) + tf'(t)dt )= integral d(tf(t))dt= [tf(t)] from 0 to 3= 87
Log in to reply
This assumes that f(x) and g(x) are differentiable.
thanks nidhin,, btw i'm a vocational school student and no so smart in calculus :) do you know how can i see the solutions to the previous week problems in "challenges" ?
Log in to reply
Hi Robi,
In the weekly challenges box, there is a tab that says "solutions" every wednesday(thursday depending where you are in the world) the solutions to all of the previous weeks problems that you looked at will be revealed.
thanks,,, thats very kind of you... :)
Note that Nidhin makes the assumption that f(x),g(x) are differentiable functions. This is not a necessary assumption (not is it valid, since we can have kinks).
Hint: The direct approach is to observe that f(x) and g(x) are inverse functions of each other. Since f(x),g(x) and are continuous functions, this implies that f(x),g(x) are strictly increasing functions.